How Is the Sum Rule for Limits Proven?

  • Thread starter Thread starter Tsunoyukami
  • Start date Start date
  • Tags Tags
    Limits Sum
Click For Summary
The discussion centers on proving the Sum Rule for Limits, which states that the limit of the sum of two functions equals the sum of their limits. The proof begins with the definitions of limits for two functions, f(x) and g(x), approaching L and M, respectively. By applying the triangle inequality, it shows that the sum of the differences can be made less than any given epsilon by appropriately choosing delta. A comparison is made to a textbook proof that uses equal splits of epsilon, raising the question of whether uneven splits can also be valid. The conclusion affirms that as long as the total remains less than epsilon, the proof is valid, confirming the flexibility in the approach.
Tsunoyukami
Messages
213
Reaction score
11
Prove the Sum Rule for Limits

$$\lim_{x\to a} [f(x) + g(x)] = \lim_{x \to a} f(x) + \lim_{x \to a} g(x) = L + M$$

Proof

Assume the following:
$$\lim_{x \to a} f(x) = L, \space\lim_{x \to a} g(x) = M$$
Then, by definition
##\forall \epsilon_1 > 0, \exists \delta_1 > 0## such that ##0<|x-a|<\delta_1 \implies |f(x)-L|<\epsilon_1##
and
##\forall \epsilon_2 >0, \exists \delta_2 > 0## such that ##0<|x-a|<\delta_2 \implies |g(x)-M|<\epsilon_2##

Choose ##\delta = \min(\delta_1, \delta_2)##.

Then ##0<|x-a|<\delta \implies |f(x)-L|<\epsilon_1## and ##|g(x)-M|<\epsilon_2##.

Notice ##|[f(x) + g(x)] - [L+M]| = |[f(x)-L] + [g(x)-M]| \leq |f(x)-L| + |g(x)-M|## by the triangle inequality. Then ##|[f(x) + g(x)] - [L+M]| \leq |f(x)-L| + |g(x)-M| < \epsilon_1 + \epsilon_2 = \epsilon##.

Then ##|[f(x) + g(x)] - [L+M]| < \epsilon##. Therefore we conclude

$$\lim_{x\to a} [f(x) + g(x)] = \lim_{x \to a} f(x) + \lim_{x \to a} g(x) = L + M$$



I was comparing my proof (the one above) to the proof offered on page 93 of James Stewart's Calculus (6e) and noticed that instead of ##\epsilon_1## and ##\epsilon_2## he uses ##\frac{\epsilon}{2}## for each of these so that their sum is ##\epsilon##.

Is there anything incorrect with my method? I figure that my approach with two different possible values of ##\epsilon_i## for each of the limits is just a more generalized way of saying the same thing - but the other proofs I've looked at online all use ##\frac{\epsilon}{2}## - is there any reason for this?

Thanks in advance!
 
Physics news on Phys.org
Tsunoyukami said:
Prove the Sum Rule for Limits

$$\lim_{x\to a} [f(x) + g(x)] = \lim_{x \to a} f(x) + \lim_{x \to a} g(x) = L + M$$

Proof

Assume the following:
$$\lim_{x \to a} f(x) = L, \space\lim_{x \to a} g(x) = M$$
Then, by definition
##\forall \epsilon_1 > 0, \exists \delta_1 > 0## such that ##0<|x-a|<\delta_1 \implies |f(x)-L|<\epsilon_1##
and
##\forall \epsilon_2 >0, \exists \delta_2 > 0## such that ##0<|x-a|<\delta_2 \implies |g(x)-M|<\epsilon_2##

Choose ##\delta = \min(\delta_1, \delta_2)##.

Then ##0<|x-a|<\delta \implies |f(x)-L|<\epsilon_1## and ##|g(x)-M|<\epsilon_2##.

Notice ##|[f(x) + g(x)] - [L+M]| = |[f(x)-L] + [g(x)-M]| \leq |f(x)-L| + |g(x)-M|## by the triangle inequality. Then ##|[f(x) + g(x)] - [L+M]| \leq |f(x)-L| + |g(x)-M| < \epsilon_1 + \epsilon_2 = \epsilon##.

Then ##|[f(x) + g(x)] - [L+M]| < \epsilon##. Therefore we conclude

$$\lim_{x\to a} [f(x) + g(x)] = \lim_{x \to a} f(x) + \lim_{x \to a} g(x) = L + M$$
I was comparing my proof (the one above) to the proof offered on page 93 of James Stewart's Calculus (6e) and noticed that instead of ##\epsilon_1## and ##\epsilon_2## he uses ##\frac{\epsilon}{2}## for each of these so that their sum is ##\epsilon##.

Is there anything incorrect with my method? I figure that my approach with two different possible values of ##\epsilon_i## for each of the limits is just a more generalized way of saying the same thing - but the other proofs I've looked at online all use ##\frac{\epsilon}{2}## - is there any reason for this?

Thanks in advance!

If you had started with a statement of what you want to prove, you would have written:

To prove: Given ##\epsilon > 0##, show there is ##\delta > 0## such that if ##0<|x-a|< \delta## then ##|(f(x)+g(x)) - (L+M)| < \epsilon##. And that is why you would pick the intermediate values in your argument so it all comes out less than ##\epsilon## at the end.

Your proof should start with "Suppose ##\epsilon > 0##" Then give your argument.
 
Oh, I see so it would be something like this:

Proof
Let ##\epsilon>0## be given. We must find ##\delta > 0## such that ##0<|x-a|<\delta \implies |f(x)+g(x) -(L+M)|<\epsilon##. By applying the triangle inequality we can write ##|f(x)+g(x) - (L+M)| \leq |f(x)-L| + |g(x)-M|##.

I will pause the proof here momentarily. The proofs I have seen have continued by letting each of these terms be less than ##\frac{\epsilon}{2}## - my question is whether or not I could let one of them be less than ##\frac{\epsilon}{3}## and the other be less than ##\frac{2\epsilon}{3}## - the sum of these terms is still ##\epsilon## so this should still be valid correct? I will attempt to complete the proof using these values.

Getting back to the main body of the proof:

Let ##|f(x) - L|<\frac{\epsilon}{3}## and ##|g(x)-M|<\frac{2\epsilon}{3}##. Since ##\epsilon>0## it follows that both ##\frac{\epsilon}{3}>0## and ##\frac{2\epsilon}{3}>0##. Since both the limit of ##f(x)## and ##g(x)## exist as ##x## approaches ##a## there exist ##\delta_1## and ##\delta_2## satisfying the following conditions:

$$0<|x-a|<\delta_1 \implies |f(x)-L|<\frac{\epsilon}{3}$$
$$0<|x-a|<\delta_2 \implies |g(x)-M|<\frac{2\epsilon}{3}$$

Choose ##\delta=\min(\delta_1, \delta_2)##. If ##0<|x-a|<\delta## then ##0<|x-a|<\delta_1## and ##0<|x-a|<\delta_2## so ##|f(x)-L|<\frac{\epsilon}{3}## and ##|g(x)-M|<\frac{2\epsilon}{3}##.

Then ##|f(x)+g(x) - (L+M)| \leq |f(x)-L| + |g(x)-M| < \frac{\epsilon}{3} + \frac{2\epsilon}{3} = \epsilon##.

This completes the proof.
What was bothering me was that every proof I saw relied upon splitting the two terms equally into ##\frac{\epsilon}{2}## when I figured that it should be possible to split ##\epsilon## into two uneven terms.

Does the above constitute a valid proof?
 
Yes. It doesn't matter how you break it up as long as the total parts add to less than ##\epsilon##.
 
  • Like
Likes 1 person
Question: A clock's minute hand has length 4 and its hour hand has length 3. What is the distance between the tips at the moment when it is increasing most rapidly?(Putnam Exam Question) Answer: Making assumption that both the hands moves at constant angular velocities, the answer is ## \sqrt{7} .## But don't you think this assumption is somewhat doubtful and wrong?

Similar threads

  • · Replies 5 ·
Replies
5
Views
2K
  • · Replies 6 ·
Replies
6
Views
2K
Replies
32
Views
3K
  • · Replies 2 ·
Replies
2
Views
2K
  • · Replies 9 ·
Replies
9
Views
3K
  • · Replies 5 ·
Replies
5
Views
2K
Replies
16
Views
2K
  • · Replies 6 ·
Replies
6
Views
2K
  • · Replies 8 ·
Replies
8
Views
2K
  • · Replies 10 ·
Replies
10
Views
2K